Retirement pre-test

¡Supera tus tareas y exámenes ahora con Quizwiz!

Which of the following statements accurately reflect the characteristics of a Section 457 plan? I. Benefits taken as periodic payments are treated as ordinary income for taxation. II. Lump-sum distributions are eligible for 5-year and/or 10-year averaging. III. Deferred amounts are subject to Social Security and Medicare taxes at the later of: performance of services or employee becomes vested in the benefits. IV. Income tax withholding is not required until funds are actually received, not constructively received. V. Cannot exceed the smaller of $19,500 or 100% includible compensation in 2021. A. I, III and V only. B. II, IV and V only. C. I, II, IV and V only. D. I, II, III, IV and V.

A. I, III and V only. There are no special tax advantages provided for 457 plans distributed in a lump-sum. Income tax withholding is required once the benefits are constructively received, even if not actually received.

Kent Reeder, age 52, works as the administrator and curator at the Museum of Antique Manuscripts, a not-for-profit organization in Metropolitan Center. He has worked there 18 years and began contributing to the 403(b) plan 12 years ago but skipped contributing last year. He earns $85,000 a year. He has asked you to maximize his contribution. Which of the following is/are TRUE? I. He may contribute $19,500 plus $6,500 for age 50+ catch-up, plus $3,000 long service catch-up. II. He may not contribute to the long-service catch-up this year due to omitting a contribution last year. III. He may contribute $19,500 plus $6,500 age 50+ catch-up. IV. He may not participate in both the long service catch-up and the age 50+ catchup the same year. V. He is not eligible for the long service catch-up. A. III and V only. B. II only. C. I, III and IV only. D. I and III only.

A. III and V only. He is not eligible for the long service catch-up because the museum is not a Health, Education, Religious (HER) organization. The maximum contribution limits for 2021 are $19,500 plus the age 50+ catch-up of $6,500.

June and Bud, both 40 years old, are not covered by a qualified retirement plan. Bud, trying to maximize their IRA deduction, put $12,000 into an IRA with June as the beneficiary on December 15 of the current year. What best describes the result of this transaction? A. June and Bud receive a tax deduction for the entire $12,000 because both spouses are eligible to contribute $6,000 to the IRA. B. Bud receives a tax deduction for $6,000 and a 6% penalty for over-contribution on the other $6,000. C. Next year Bud will receive a $6,000 deduction, in addition to the $6,000 deduction for this year. D. Bud receives a tax deduction for $6,000 and is considered to have made a non-deductible contribution of the other $6,000.

B. Bud receives a tax deduction for $6,000 and a 6% penalty for over-contribution on the other $6,000. This question indicates an IRA in only Bud's name. Maximum contribution is $6,000 in 2021, plus any applicable catch-up provisions. Amounts contributed over that level are considered excess contributions and subject to a 6% penalty until taken out. The 6% penalty could have been avoided if the excess contribution was withdrawn prior to the original filing deadline without extension.

Which of the following individuals are "key employees" as defined by the Internal Revenue Code? I. A more-than-5% owner of the employer business. II. An employee who received compensation of more than $120,000 from the employer. III. An officer of the employer who received compensation of more than $190,000. IV. A 1% owner of the employer business having annual compensation from the employer of more than $55,000. A. I and II only. B. I and III only. C. II and IV only. D. I, II and IV only.

B. I and III only A key employee is an individual who (1) owns more than 5% of the business, (2) is an officer with compensation greater than $185,000 (2021), or (3) owns at least 1% of the business and has compensation greater than $150,000. I. and III. are defined key employees. II. defines a highly compensated employee, not a key employee. IV. should state that compensation was more than $150,000.

Qualified retirement plans have which of the following characteristics? Employees with one year of service and attained age 21 must be participants in the plan. Fund earnings are usually not taxed until distributions are received by the employee. All lump-sum distributions are eligible for five-year forward averaging tax treatment. Employer contributions to the plan are deductible in the year they are made (or deemed made), subject to IRC Section 415 limits. A. I and III only. B. II and IV only. C. I and II only. D. III and IV only.

B. II and IV only Maximum waiting period for qualified plans is two years (except for SEPs [employer sponsored tax advantaged plan] which can have a 3-year waiting period). No lump sum distributions are eligible for 5-year averaging after December 31, 1999.

Kipton is an executive with BigRock. As part of his compensation, he receives 10,000 shares of restricted stock today worth $20 per share. The shares vest two years from today, at which point the stock is worth $30 per share. The vesting schedule is a 2-year cliff schedule. Kipton holds the stock for an additional 18 months and sells at $45 per share. Which of the following is correct? A. The grant of stock is taxable to Kipton today. B. The value of the shares is taxable to Kipton when the stock vests. C. If Kipton were to make an 83(b) election, he would have converted $30 of the gain from ordinary to capital. D. When Kipton sells the stock for $45 per share, his basis is $30 regardless of whether he files an 83(b) election.

B. The value of the shares is taxable to Kipton when the stock vests. Choice a is not correct because the stock is forfeitable. Choice c is not correct because it would have converted $10, not $30. Choice d is not correct, because the basis would be different.

Wilber receives incentive stock options (ISOs) with an exercise price equal to the FMV at the date of the grant of $15. Wilber exercises these options 3 years from the date of the grant when the FMV of the stock is $35. Wilber then sells the stock 3 years after exercising for $45. Which of the following statements is (are) true? A. At the date of grant, Wilber will have ordinary income equal to $15. B. At the date of exercise, Wilber will have W-2 income of $20. C. At the date of sale, Wilber will have long-term capital gain of $30. D. Wilber's employer will have an income tax deduction related to the exercise of the option by Wilber.

C. At the date of sale, Wilber will have long-term capital gain of $30. Choice a is not correct as there is no income at the date of grant because the strike price equals the FMV. Choice b is not correct as there is no regular tax for ISOs. Choice d is not correct because the employer will not have an income tax deduction. Grant 15 Exercise @35 (AMT adjustment of $20) Sale $45 $10 LTCG from gain (exercise to sale) $20 LTCG from exercise Total $30 LTCG

Angelo's Bakery has 105 employees. 90 of the employees are nonexcludable and 15 of those are highly compensated (75 are nonhighly compensated). The company's qualified profit sharing plan benefits 8 of the highly compensated employees and 40 of the nonhighly compensated employees. Does the profit sharing plan sponsored by Angelo's Bakery meet the coverage test? A. Yes, the plan meets the average benefits percentage test. B. Yes, the plan meets the general safe harbor test. C. Yes, the plan meets the ratio percentage test. D. Yes, the plan meets ratio percentage test and the general safe harbor test.

C. Yes, the plan meets the ratio percentage test. The plan meets the ratio percentage test. The percentage of NHC employees covered by the plan is 53.33% and the percentage of HC employees covered by the plan is 53.33%. The ratio percentage of the NHC employees covered by the plan compared to the ratio percentage of the HC employees covered by the plan is 100% (53.33% / 53.33%) which is greater than the ratio requirement of at least 70%.

You have been hired to analyze the retirement prospects of Tom and Jerri Ruhn. It has been determined they need a retirement capital account of $2,750,000 at retirement which will occur in 30 years. They expect to live in retirement for 35 years. They are anxious to start a savings program to meet this goal. They anticipate an average after-tax rate of return equal to 7%. They are planning on 5% annual inflation. What level of savings put away at the end of each year will provide the Ruhn family with their desired retirement fund? $27,208 $29,113 $67,787 $68,884

B. $29,113 The client has given us the capital account they want at retirement in 30 years. If it said in today's dollars or the equivalent, then you would account for inflation. That information was a distractor in this question. N = 30 I = 7 PV = 0 PMT=29,113 FV = 2,750,000

Which of the following is true regarding Sensitivity Analysis? A. It is a mathematical tool that utilizes a probabilistic distribution of returns and their effects on an individual's retirement plan. B. It is the changing of variable assumptions to determine the effects to the retirement plan. C. It is a test used to determine if a plan passes the ADP test. D. It is a tool to determine if a Roth IRA distribution is taxable.

B. Sensitivity Analysis is the changing of variable assumptions to determine the effects to the retirement plan. This is typically done by rotating a variable toward increased risk. For example, if inflation assumption is 3%, what would happen to the plan if it actually turns out to be 3.5% or 4%. Monte Carlo Analysis is a mathematical tool that utilizes a probabilistic distribution of returns and their effects on an individual's retirement plan.

RCM Incorporated sponsors a qualified plan that requires employees to meet one year of service and to be 21 years old before being considered eligible to enter the plan. Which of the following employees are not eligible? I. Donald, age 18, who has worked full-time with the company for 3 years. II. Rachel, age 22, who has worked full-time with the company for 6 months. III. Randy, age 62, who has worked 500 hours per year for the past 6 years. IV. Theodore, age 35, who has worked full-time with the company for 10 years. A. IV only. B. I and II only. C. III and IV only. D. I, II and III only.

D. I, II, and III only RCM, Inc. cannot exclude anyone who has attained age 21 and has completed one year of service with the company with 1,000 hours during that year. Plan years beginning after December 31, 2020, SECURE Act provides for employee who have worked at least 500 hours per year with the employer for at least three consecutive years and has met the age requirement (age 21) by the end of the three consecutive year period. For determining whether the three-consecutive-year period has been met, the 12-month periods beginning before Jan. 1, 2021 will not be taken into account. Randy will be eligible in 2024 after three consecutive years.

Match the following statement with the type of retirement plan which it most completely describes: "A qualified plan which allows employee elective deferrals of 100% of includible salary and has a mandatory employer match" is... A Profit sharing plan. A Money purchase plan. A SIMPLE 401(k). A Defined benefit plan.

A SIMPLE 401(k) Profit sharing plans "A" are not contributory. Answers "B" and "D" do not permit employee elective deferrals.

Which of the following would reduce the amount needed to be saved on an annual basis in order to accumulate sufficient retirement assets? A. Accelerated inflation. B. Expansion of current lifestyle. C. Excess returns on investments over projections. D. Increased life expectancy due to medical advances.

C. Excess returns on investments over projections. Increasing inflation, expanded lifestyle, and increased life expectancy all increase the amount needed to fund retirement needs.

A SEP-IRA is a form of defined contribution plan (although not a qualified plan). Which of the following apply to BOTH the SEP-IRA and a traditional defined contribution plan? I. Employer deductions limited to 15% of covered payroll. II. Requires a definite, written, non-discriminatory contribution allocation formula. III. Contributions cannot discriminate in favor of highly compensated employees. IV. Employer contributions subject to Medicare and Social Security taxes. V. Affiliated service group rules apply. VI. Top-heavy rules do NOT apply. VII. Permissible disparity or integration is NOT allowed. A. I, II, VI, and VII only. B. II, III, IV and VI only. C. II, III and V only. D. I, II, III, V and VII only.

C. II, III, and V only Defined contribution plans have an employer deductibility limit of 25% of covered payroll. All defined contribution plans must have a written allocation formula so assets can be distributed in the mandated individual accounts. Employer contributions must bear uniform resemblance to compensation and cannot discriminate in favor of highly compensated. Employer contributions are not subject to any payroll related taxes. Top-heavy rules do apply to both. Both plans can integrate with Social Security (sometimes called permissible disparity). (Note: 5305-SEP does not allow permissible disparity.)

A client, age 74, must receive a minimum distribution from his IRA account for this year, which had a value (at the end of the prior year) of $48,000. His spouse, age 73, is the beneficiary of the IRA account. The life expectancy according to IRS tables is 14.1 years. If the client takes a $2,000 distribution on December 31st, what will be the tax penalty if any? $0 $277 $702 $1,404

$702. The $48,000 is subject to a 14.1 year life expectancy of the client, thus minimum distribution is $3,404 per year. If the client takes only $2,000, the balance ($1,404) is subject to a 50% excise tax or $702 penalty.

Which of the following is not a qualified retirement plan? ESOP. 401(k) plan. 403(b) plan. Target benefit plan.

403(b) plan A 403(b) plan is a tax-advantaged plan, not a qualified plan. All of the others are qualified plans.

Which of the legal requirements apply to defined benefit pension plans? I. Each participant must have a separate account to hold assets. II. An actuary is needed to calculate the minimum funding level. III. Retirement benefits can be adjusted based on sponsor profits. IV. The benefits in most traditional defined benefit plans are protected, within certain limitations, by federal insurance provided through the Pension Benefit Guaranty Corporation (PBGC). A. I and III only. B. II and IV only. C. I, II and IV only. D. II, III and IV only.

B. II and IV only. Statement "I" applies only to defined contribution plans. Statement "III" describes a profit-sharing plan.

Ernest converted his Traditional IRA to a Roth IRA on Dec 15, 2018. He was 35 years of age at the time and had never made a contribution to a Roth IRA. The conversion was in the amount of $60,000 ($10,000 of contributions and $50,000 of earnings). Over the years he has also made $15,000 in contributions. On May 15, 2021 he withdrew the entire account balance of $100,000 to pay for a 1 year trip around the world. Which of the following statements is true? A. $25,000 of the distribution will be subject to income tax and $85,000 of the distribution will be subject to the 10% early withdrawal penalty. B. $25,000 of the distribution will be subject to income tax and the 10% early withdrawal penalty. C. Some of the distribution will be taxable but the entire distribution will be subject to the 10% early withdrawal penalty. D. None of the distribution will be taxable nor will it be subject to the 10% early withdrawal penalty.

A. $25,000 of the distribution will be subject to income tax and $85,000 of the distribution will be subject to the 10% early. Roth distributions are tax free if they are made after 5 years and because of 1)Death, 2)Disability, 3) 59.5 years of age, and 4)First time home purchase. He does not meet the five year holding period or one of the exceptions. His distribution does not received tax free treatment. The treatment for a non-qualifying distribution allows the distributions to be made from basis first, then conversions, then earnings. His basis will be tax free. The conversion is also tax free since we paid tax at the time of the conversion on those earnings. The remaining earnings since establishment of the Roth are $25,000 (100,000 - $15,000 in basis - $60,000 in conversions) and will be taxed. The 10% penalty does apply to this distribution since he does not qualify for any of the exceptions to the penalty. The contributions escapes penalty but the conversions and earnings of $85,000 are subject to the 10% early withdrawal penalty. Remember that in order for the conversions to escape the 10% early withdrawal penalty the distribution must occur after a 5 year holding period beginning Jan 1 in the year of conversion or meet one of the 10% early withdrawal exceptions. Summary: $60,000 paid tax at conversion. Subject to penalty $15,000 in contributions no tax, no penalty $25,000 earnings. Taxable and subject to penalty Took out 100,000; $25,000 is taxable, $85,000 subject to penalty

Mary Anne has AGI of $1,000,000 (which is all comprised of earned income). She is single and age 55. She is not an active participant in her employer's qualified plan. Which of the following statements best describes her options? She can contribute to a Traditional IRA and deduct her contribution. She can contribute to a Traditional IRA but not deduct her contribution. She can contribute to a Roth IRA. She cannot contribute to a Traditional IRA or Roth IRA

She can contribute to a Traditional IRA and deduct her contribution. She can contribute and deduct her contribution to a Traditional IRA since she is not an active participant and therefore not subject to an AGI limitation. She is unable to contribute to a Roth IRA because she is above the AGI limitation of $125,000 - $140,000 (2021).

Brisco, age 51, is the Executive VP of sales at Doggie Daycare (DD). His base salary is $300,000 with a potential bonus of 50%. Brisco is a participant in his employer's 401(k) plan and always defers the maximum amount. The DD 401(k) plan has the following features and characteristics: Includes a Roth account that is not a safe harbor plan, but has a 50% match up to 4%. The ADP for the NHC is 4.5%. The plan has $3 million in assets that are managed by two asset management firms. DD also sponsors a defined benefit plan that provides a benefit based on years of service and final salary. The DD DB plan provides for 1.5 percent per year of service for the first 20 years and 2 percent for years above 20, up to a max of 35 years. On the weekends, Brisco paints murals. His entity, Wall Works LLC (WW), is a single owned LLC taxed as a disregarded entity. Brisco would like to establish a retirement plan for the income that he earns in WW. He expects to earn $60,000 ever year in WW and wants to know what the best retirement plan is for his business. Which plan would you recommend for him? A. SIMPLE B. Defined Benefit Plan C. 401(k) plan D. SEP

D. SEP Neither a SIMPLE nor a 401(k) plan will work because he is already deferring $18,850 (6.5% (4.5% + 2%) times $290,000) to his 401(k) plan. Therefore, the choices are a defined benefit plan (expensive) or a SEP, which is extremely easy to set up and one that he can contribute around $11,000 to annually. Instructor note: He is currently contributing 6.5% of salary capped at 290,000 for 2021, which is 18,850. The most he can contribute to multiple 401k plans is limited to 19,500. A 401k is rather expensive with ongoing filing for him to contribute $650 (19,500 -18,850). The most he can do is a match as the employer, the 401k did not state it had a Profit sharing component (nor did it state it was a solo 401k). A SEP is much cheaper to set up and has no annual report requirements like the 401k. He was be able to contribute as an employer to the SEP, which will be based on 20% of net income, around the $11,000 amount (without doing the full self-employment calculation).

Which of the following will be subject to a 10% early withdrawal penalty? Sylvia, age 56, retired from Marshall Corporation. She takes a $125,000 distribution from the Marshall Corporation Defined Contribution Retirement Plan to pay for living expenses until she is eligible for Social Security. Terry quits Shoe Shine Company at age 48. He begins taking equal distributions over his life expectancy from his qualified plan after separating from service. The annual distribution is $2,000. Kevin's wife had a baby 10 days ago. He withdrew $2,000 from his IRA to cover costs for baby furniture, diapers and carseats. Edward, age 40, takes a $40,000 distribution from his profit-sharing plan to pay for his son's college tuition.

D. Edward, age 40, takes a $40,000 distribution from his profit-sharing plan to pay for his son's college tuition. There is no provision for a distribution without penalty under this circumstance. Edward is only 40 and education withdrawals are allowed in IRAs, not from qualified plans. SECURE Act 2019 added penalty free withdrawals up to $5,000 taken within 12 months of birth or legal adoption.


Conjuntos de estudio relacionados

CompTIA Network+ N10-006 Chapter 2 Quiz

View Set

Monitoring for Health Problems (9)

View Set

Trail Guide to the body Chapter 4

View Set

Current Digital Forensics Tools (Module 6 Review) - [Computer Forensics]

View Set